Đến nội dung

Kamii0909 nội dung

Có 155 mục bởi Kamii0909 (Tìm giới hạn từ 21-04-2020)



Sắp theo                Sắp xếp  

#662811 Có số nguyên dương n nào thỏa mãn ${3^n} + 2003\,\,...

Đã gửi bởi Kamii0909 on 23-11-2016 - 14:42 trong Số học

Ta sẽ chứng minh không tồn tại.
Thật vậy ta phải có
$8|3^n+3$
Với $n=2k,8|3^n-1$
Với $n=2k+1,8|3^n-3$
Từ đó có đpcm.



#662810 Tìm n để trong $S_{n}$ không có số chính phương nào

Đã gửi bởi Kamii0909 on 23-11-2016 - 14:30 trong Số học

Bổ đề:Cho 2 số nguyên dương $a>b$
Nếu $\sqrt{a}-\sqrt{b} \geq 1$ thì $[a,b)$ có ít nhất 1 số chính phương.
*Lưu ý:Điều ngược lại là sai. Ví dụ $a=8,b=10$.
Như vậy ta phải có
$\sqrt{n}-\sqrt{\frac{n}{2}} <1$
Bằng biến đổi tương đương ta nhận được
$\sqrt{n} < \sqrt{2}(\sqrt{2}+1)$
Hay $n \leq 11$.
Thử trực tiếp có $n=9$ thỏa mãn.



#662754 Cho tam giác ABC với I là tâm đường tròn nội tiếp

Đã gửi bởi Kamii0909 on 22-11-2016 - 21:16 trong Hình học

Nguyên gốc như sau.
$\frac{AI}{AD}=\frac{IH}{ID}$



#662752 Cho tam giác ABC với I là tâm đường tròn nội tiếp

Đã gửi bởi Kamii0909 on 22-11-2016 - 21:15 trong Hình học

Tam giác DCA và DHC đồng dạng nên $\frac{DC}{DH}=\frac{CA}{HC}$. Tam giác ACH có CI là phân giác trong góc C nên $\frac{CA}{HC}=\frac{AI}{IH}$. Vậy $\frac{DC}{DH}=\frac{CA}{HC}=\frac{AI}{IH}$

Tks bác. Lần đầu e thấy cách thế này. Tại có 1 đẳng thức gần giống thế làm bằng định lý sin nên e bắt chước.



#662736 Cho tam giác ABC với I là tâm đường tròn nội tiếp

Đã gửi bởi Kamii0909 on 22-11-2016 - 20:42 trong Hình học

Bác Kamii0909 đã đến chỗ $\frac{AI}{IH}=\frac{DC}{DH}$ thì chỉ cần tam giác DCH và DAC đồng dạng nữa là OK, không cần định lý sin.

Em đã thử và nó ko ra bác ạ.



#662735 Điều kiên đồng quy của 3 đường thẳng

Đã gửi bởi Kamii0909 on 22-11-2016 - 20:36 trong Hình học

$d$ là đường thẳng kẻ từ $P$ vuông góc với $AD$ hỏ cá?

d là $ PS $ .Đã sửa.



#662731 Điều kiên đồng quy của 3 đường thẳng

Đã gửi bởi Kamii0909 on 22-11-2016 - 20:28 trong Hình học

Cho $\Delta ABC$ nhọn nội tiếp $(O)$ có $AB<AC$. Phân giác góc $\widehat{A}$ cắt $(O)$ tại $D$. Từ 1 điểm $P$ trên $AB$ kẻ $PS$ vuông góc với $AD$. $PD$ cắt $(O)$ tại $G$.$SB,SG$ cắt $(O)$ tại $M,N$.Chứng minh rằng điều kiện cần và đủ để $BD,PS,AG$ đồng quy là tứ giác $ABMN$ điều hòa.



#662729 Cho tam giác ABC với I là tâm đường tròn nội tiếp

Đã gửi bởi Kamii0909 on 22-11-2016 - 20:16 trong Hình học

Bài này là mở rộng của IMO 2010.
Gọi $SD$ giao $BC$ tại $K$.
Dễ có $DB^2=DC^2=DI^2=DK.DS$ nên $\Delta{DIK}$~$\Delta{DSI}$
Từ đó có $\widehat{DIK}=\widehat{DSI}=\widehat{DAF}$ hay $AF//IK$.
Gọi $AD$ cắt $BC$ tại $H$.
Áp dụng định lý Menelaus vào tam giác $IFH$ và định lý Thales ta phải chứng minh
$\frac{AI}{IH} =\frac{DI}{DH}=\frac{DC}{DH}$
Áp dụng định lý sin vào tam giác $AIC$, $IHC$ và $DHC$ ta có
$\frac{AI}{IH}=\frac{AI}{IC}.\frac{IC}{IH}= \frac{\sin IHC}{\sin IAC}=\frac{\sin DHC}{\sin DCH}=\frac{DC}{DH}$



#662553 Chứng minh $I,E,F$ thẳng hàng .

Đã gửi bởi Kamii0909 on 20-11-2016 - 20:54 trong Hình học phẳng

Đề có đúng ko bạn. Mình vẽ nó sai



#662540 Chứng minh $MK=MA$

Đã gửi bởi Kamii0909 on 20-11-2016 - 19:52 trong Hình học

Ta có $MK^2=MO^2-R^2=MH^2+HO^2$
$MA^2=MH^2+HA^2$
Gọi $OA$ cắt $BC$ tại $Q$. Khi đó $HA=HQ$
Khi đó ta phải chứng minh
$HO^2-HA^2=R^2$
$\Leftrightarrow (HO-HQ)(HO+HA)=R^2$
$\Leftrightarrow OQ.OA=OB^2$
Đẳng thức cuối là cơ bản.



#662524 Chứng minh $IM$, $IN$ tiếp xúc với $(AMN)$

Đã gửi bởi Kamii0909 on 20-11-2016 - 16:48 trong Hình học

Bài này có khá nhiều cách giải.
1.Dễ thấy $\Delta AMN $ cân.
Gọi giao điểm $(O)$ và $(ADE)$ là $K$.
Khi đó $K$ cũng thuộc $(AMN)$.
Dễ có $AI,KH$ cắt nhau tại 1 điểm trên $(AMN)$ là $X$.
Ta có $\frac{AM}{AN}=\frac{XM}{XN}$ nên AMXN điều hòa.
2. Theo định lý về tâm đẳng phương
$AK,BC,DE$ đồng quy tại $P$.
Gọi $Q$ là giao điểm $AH$ và $KI$.
Do $AO$,$AH$ đẳng giác nên $Q$ thuộc $(AMN)$.
$AQ$ cắt $BC$ tại $Y$.
Dễ có $(PY,BC)=-1$.
Chiếu hàng này lên $(AMN)$ thì $KMQN$ điều hòa.
Tư đó $KQ$ và $AX$ cắt nhau tại $I$ là giao điểm 2 tiếp tuyến của $(AMN)$.



#662505 Đề Thi HSG Toán TP Hải Phòng ( bảng không chuyên ) năm 2016-2017

Đã gửi bởi Kamii0909 on 20-11-2016 - 11:23 trong Thi HSG cấp Tỉnh, Thành phố. Olympic 30-4. Đề thi và kiểm tra đội tuyển các cấp.

sai rồi bạn nhé
thay thử lại thì P=-1/3 và đó cũng là đáp án đúng

Xin lỗi bạn lúc đó mình hơi vội.
Mình xin đưa cách giải như sau.
Ta chứng minh $VP \leq -1/3$
$\Leftrightarrow \sum \frac{4}{x+2} - \sum \frac{3x^2}{2} \geq 26/3$
Ta chứng minh bất đẳng thức phụ
Với $0 \leq x \leq 1$ thì
$\frac{4}{x+2} - \frac{3x^2}{2} \geq \frac{-13x}{6} +2$
$\Leftrightarrow x(x-1)(9x+14) \leq 0$
Áp dụng bất đẳng thức phụ rồi cộng vào ta có đpcm.



#662270 Chứng minh $G,G_1,G_2$ thẳng hàng.

Đã gửi bởi Kamii0909 on 17-11-2016 - 22:32 trong Hình học

Ta có $3\overrightarrow{GG_{1}}+3\overrightarrow{GG_{2}}=\sum\overrightarrow{GA_{1}}+\sum\overrightarrow{GA_{2}}=\overrightarrow0$

nên $G,G_{1},G{2}$ thẳng hàng. Cách sử dụng vecto. Bạn tham khảo




#662264 Cho các số thực $a,b,c$ thỏa mãn $a^{2}+b^{2...

Đã gửi bởi Kamii0909 on 17-11-2016 - 22:10 trong Bất đẳng thức và cực trị

v~ cả nhái :D , trích (Poland 1991) đã được giải ở đây bn: http://diendantoanho...e-6#entry660132

Cách giải của bạn hanguyen445 trong phần a không âm chính là cách giải của bài thi trên. 

Bài này khá nổi tiếng, có xuất hiện trong TLCT 10 mà bạn.




#662090 Cho các số thực $a,b,c$ thỏa mãn $a^{2}+b^{2...

Đã gửi bởi Kamii0909 on 15-11-2016 - 22:35 trong Bất đẳng thức và cực trị

Bài này có lẽ "nhái" theo 1 bài thi Châu Âu nhưng bất thành. Thành ra là 1 bài rất khó. Mình nghĩ nó là bài sau:
Cho $x,y,z$ thực thỏa mãn $x^2+y^2+z^2=2$
Chứng minh rằng $x+y+z-xyz \leq 2$.



#662053 cho a,b,c>0.CMR: $\sum \sqrt{\frac{a}...

Đã gửi bởi Kamii0909 on 15-11-2016 - 19:26 trong Bất đẳng thức và cực trị

1 cách nữa cho bài 2.
Không mất tính tổng quát giả sử $a \geq b \geq c$.
Khi đó $VP \geq \sum \frac{b^2}{b+c}$ và$ VP \geq \sum \frac{c^2}{c+a}$
Từ đó ta đi chứng minh
$\sum \frac{b^2+c^2}{b+c} \geq \sqrt \frac{2011}{2}$
Áp dụng bất đẳng thức Cauchy-Schwarz ta có $2VP \geq \frac{(\sum \sqrt {a^2+b^2})^2 }{2\sum a}$
Dễ dàng suy ra đpcm từ đây



#661756 Bài toán về tứ giác toàn phần

Đã gửi bởi Kamii0909 on 13-11-2016 - 12:03 trong Hình học

Mình vẽ hình chả thấy đồng viên ,chỉ thấy thẳng hàng : 

Từ $O$ hạ đường vuông góc đến $PQ$ nghĩa là đường kẻ từ $O$ đến vuông với $PQ$ đi qua tâm $(ABCD)$ ,hay $R$ là điểm Miquel của tứ giác $ABCD.PQ$ từ đó có $(ABQR),(ADPR)$ là các tứ giác nội tiếp nên theo Simson thì các điểm $Z,Y,X,T$ thẳng hàng

ABCD có nội tiếp đâu bạn.




#661753 $QD$ chia đôi $IH$

Đã gửi bởi Kamii0909 on 13-11-2016 - 11:49 trong Hình học

Đây là bài thi IMO 2010. Có khác nhiều cách làm.




#661744 Kỳ thi chọn đội tuyển dự thi VMO tỉnh Đồng Nai

Đã gửi bởi Kamii0909 on 13-11-2016 - 11:11 trong Thi HSG cấp Tỉnh, Thành phố. Olympic 30-4. Đề thi và kiểm tra đội tuyển các cấp.

Chứng minh đẳng giác có thể dùng TAB và TCA đồng dạng. D, E là trung điểm của AB, AC nên TAD và TCE đồng dạng.

Cũng giống cách của mình. Nhưng mình đang tìm cách chứng minh nhanh hơn. 




#661729 Kỳ thi chọn đội tuyển dự thi VMO tỉnh Đồng Nai

Đã gửi bởi Kamii0909 on 13-11-2016 - 09:29 trong Thi HSG cấp Tỉnh, Thành phố. Olympic 30-4. Đề thi và kiểm tra đội tuyển các cấp.

Câu hình còn có thể chứng minh như thế này :
Gọi tiếp tuyến kẻ từ $A$ cắt $BC$ tại $T$ ; tâm ngoại tiếp $(BAC),(ADE)$ lần lượt là $H,G$ dễ thấy $A,G,H$ thẳng hàng do đó $TA$ là tiếp tuyến chung của $(ABC),(ADE)$. Nên $T$ nằm trên trục đẳng phương của $(AED),(BCD),(BCE)$ hay $DP,QE,BC$ đồng qui tại $T$ .Ta có :
$\widehat{ACB}=\widehat{TQB}=\widehat{TAB}$ suy ra tứ giác $AQBT$ nội tiếp
Mặt khác:
$\widehat{AQP}=\widehat{ADP}=\widehat{ATD}+\widehat{TAD}$
Và $\widehat{APQ}=\widehat{AEQ}=\widehat{TAB}+\widehat{BAQ}$
Kết hợp với tia $TD$ và tia $TQ$ đẳng giác trong $\widehat{ATB}$ nên $\widehat{ATD}=\widehat{BAQ}$

Cái khó của bài toán chính là $TD$ và $TQ$ đẳng giác. Bạn có hướng nào ngắn gọn chứng minh cái này không?



#661703 Phương trình nghiệm nguyên

Đã gửi bởi Kamii0909 on 12-11-2016 - 23:51 trong Số học

Mình lý luận không được chắc chắn lắm có gì sai sót mong bạn chỉ giúp.

đặt $a=x+y,b=xy$

thì $a^2|a^3-3ab-b^2$ hay $a^2|3ab+b^2$

Từ đó $ka^2-3ab-b^2=0$

Coi đây là phương trình bậc 2 ẩn $a$ thì 

$\Delta =b^2(4k+9)$ nên $4k+9$ phải là số chính phương lẻ.

Từ đó tính được $k=m^2+m-2$

thay vào công thức nghiệm được $a= \frac{b}{m-1}$ hoặc $a= \frac{b}{m+2}$

Cả 2 đều chứng tỏ $a|b$ hay $x+y|xy$ nghĩa là $xy=p(x+y)$

Từ đề bài thì tử số phải không âm hay

$x+y \geq 3p+p^2$

Nếu $x=y$ thì $p= \frac{x}{2}$ thay vào có $x \leq 2$

$x=y=1$ KTM, $x=y=2$ TM.

TH còn lại xử lí như trên.




#661696 Cho $a^2+b^2+(a-b)^2=c^2+d^2+(c-d)^2$. CMR: $a^4+b^4+(a-b)^4=c...

Đã gửi bởi Kamii0909 on 12-11-2016 - 23:26 trong Số học

Có hằng đẳng thức sau 

$a^4 +b^4 +(a\pm b)^4=2(a^2\pm ab+b^2)^2$

Hoàn toàn dễ dàng chứng minh đẳng thức trên dựa vào đây.




#661694 Kỳ thi chọn đội tuyển dự thi VMO tỉnh Đồng Nai

Đã gửi bởi Kamii0909 on 12-11-2016 - 23:19 trong Thi HSG cấp Tỉnh, Thành phố. Olympic 30-4. Đề thi và kiểm tra đội tuyển các cấp.

Bài hình không khó lắm nhưng khá hay.
Có $A$ là tâm vị tự biến $(ADE)$ thành $(ABC)$ nên $(ADE)$ tiếp xúc $(ABC)$ tại A.
Từ đó theo định lý về tâm đẳng phương thì
$BC,EQ,PQ$ và tiếp tuyến chung tại $A$ của 2 đường tròn $(ADE)$ và $(ABC)$ đồng quy tại S.
Ta có $(ED,EQ)=(SB,SQ)=(AB,AQ)(mod \pi )$ hay ASBQ nội tiếp.
Dễ thấy $(BD,BQ)=(SA,SQ)(mod \pi )$ và $(DB,DQ)=(EA,EQ)=(AS,AQ)(mod \pi )$
nên $\Delta BDQ \sim \Delta SAQ$
Suy ra $\frac{BD}{BQ} = \frac{SA}{SQ} = \frac{AD}{BQ}$
Mà $(AS,AD)=(QS,QB)(mod \pi )$ nên $\Delta ASD \sim \Delta QSB$
Có $(AD,DP)=(QB,BC)=(EQ,EA)=(PQ,PA)=(QA,QP)(mod \pi )$
Hệ thức trên chứng tỏ $AQ=AP$.




#661639 Giải pt nghiệm nguyên $k^{2}-2016=3^{n}$

Đã gửi bởi Kamii0909 on 12-11-2016 - 14:22 trong Đại số

n le

n lẻ thì sao đặt như v đc

Đã chứng minh n chẵn mà bạn



#661638 Chứng minh rằng $\sum \frac{x^{4}+y^{4...

Đã gửi bởi Kamii0909 on 12-11-2016 - 14:16 trong Số học

Bài này nên đưa vào mục bất đẳng thức chứ nhỉ.
Có thể giải bằng Holder như sau(không hay lắm)
$(x^4 +y^4)^3(1+1) \geq (x^3+y^3)^4$
và $(x^3+y^3)(1+1)(1+1) \geq (x+y)^3$
Từ đó ta có
$\frac{x^4+y^4}{x^3+y^3} \geq \frac{x+y}{2}$
Cộng lại ta cũng có đpcm